Berechnen Sie die Wasserdurchflussrate durch die Öffnung

Ich bin nicht sehr gut in Strömungsphysik und brauche etwas Hilfe. Stellen Sie sich den folgenden Aufbau mit Wasser vor einer Wand mit einer Öffnung am Boden vor:

Fließendes Bild

Wie berechne ich den Wasserdurchfluss Q ?. Ich habe einige Nachforschungen angestellt und festgestellt, dass ich den Druck über der Öffnung (Öffnung) (teilweise) berechnen muss. Aber ich kenne den Druck auf der Rückseite der Öffnung nicht. Kann man das irgendwie lösen?

Hinweis: Ich sage nicht "Bitte geben Sie mir die Lösung, ich bin faul" . Ich will es selbst herausfinden. Da ich in diesem Fall aber nur Formeln zur Berechnung des Druckabfalls gefunden habe, kann ich sie nicht zur Lösung des Problems verwenden. Deshalb wende ich mich Ihnen zu, um zu sehen, ob es einen anderen Weg gibt, dieses Problem zu lösen.

Update: Der "Tank", der das Wasser enthält, ist tatsächlich a big lake, und die Öffnung ist, wie weit sich das Wassertor geöffnet hat. Ich muss sehr genau berechnen, wie viel Wasser durch die Öffnung fließt.

Wenn das Loch klein genug ist, kann der Druck darauf als konstant angesehen werden ( D H ), dann ist der Druck einfach die Fülldichte mal H mal Schwerkraft.
Ach, dem ist leider nicht so. variiert und kann in einigen Fällen dfast gleich sein .h
Ich bin mit den knappen Abstimmungen zu dieser Frage nicht einverstanden. Dies ist ein Beispiel für "Welche Tools/Physik brauche ich, um dieses Problem zu lösen?", von dem wir uns alle einig waren, dass es im Meta-Thread zum Thema gehört
Ist der Tank groß genug dafür H bleibt konstant? (nicht unbedingt das D H , es kann ein sehr breiter Tank sein)
Der Tank ist eigentlich a big lake, und das ist das Schleusentor im Kraftwerk. Ich muss die Wasserdurchflussmenge sehr genau berechnen, je nachdem, wie weit sich die Luke geöffnet hat (die Entfernung D )
Solange das Loch relativ zum gesamten Tank "klein" ist, ist die Berechnung des Drucks ziemlich einfach. Der Druck am Boden des Tanks ist gleich dem Gewicht der darüber liegenden Flüssigkeit. Das wäre also D*g*h. An der Spitze des Lochs steht (hd) statt h. Der durchschnittliche Druck wäre gleich dem Druck am Mittelpunkt. Reicht das zum Weitermachen?
@Jim Ich stimme zu, aber Eric hätte viel mehr von dem zeigen können, was er tatsächlich bereits getan hat, da er anscheinend mehr Informationen hat, als er teilt.
Es tut mir leid, wenn ich nicht genügend Hintergrundinformationen zu dem Thema geliefert habe. Ich dachte daran, es ein bisschen mehr zu verallgemeinern, um vielleicht andere ähnliche Fragen zu beantworten. Die vollständige Beschreibung meines Falls ist jetzt als Update hinzugefügt.

Antworten (4)

Vermute das erstmal H ändert sich nicht sehr, weil Sie ein großes Gewässer haben (wir können diesen Zustand später lockern). Nehmen wir außerdem an, dass das Loch klein im Vergleich zur Tiefe ist ( D H ) - wir lockern das auch. Für diesen Fall ist die Antwort einfach, Sie würden die Bernoulli-Gleichungen verwenden und einfach den statischen Druck ( ρ G H ) gleich dem dynamischen Druck ( 1 2 ρ v 2 ). Dann würdest du ausziehen v und mit der Fläche multiplizieren A des Lochs zu bekommen Q , seit Q ist der Volumenstrom.

Lassen Sie uns nun die Bedingung lockern, dass D H . Da der Druck am Loch mit der Tiefe variiert, ändert sich auch die Geschwindigkeit. Sie können dies wie eine Rechenaufgabe behandeln, bei der Sie die inkrementelle Änderung der Geschwindigkeit als Funktion der Höhe berechnen. Berechnen Q , müssten Sie integrieren w v ( X ) D X für X = 0 Zu X = D . Notiz w wäre die Breite Ihres Lochs in der Seite (unter der Annahme eines quadratischen Lochs).

Sobald Sie den obigen Ausdruck erhalten ( Q als Funktion von H ), könnte man dann die Bedingung lockern H Seien Sie konstant, indem Sie das bemerken H hängt vom Volumenstrom und der Geometrie des Sees ab. Sobald du hast Q ( H ) aus dem vorherigen Schritt können Sie das zur Berechnung verwenden H ( T ) und ersetzen Sie das wieder in Ihre Gleichung aus dem vorherigen Schritt.

Bitte entschuldigen Sie, da ich seit 4 Jahren nicht mehr mit Rechnen gearbeitet habe. Ich habe ein paar Fragen: v ( X ) = 2 G X die binde ich so ein wie du es beschrieben hast, oder? Aber dann Q ( H ) als ergebnis, was meinst du mit rechnen H ( T ) ? Vielen Dank im Voraus.
@Eric Sobald Sie Q (h) erhalten haben und h nicht zeitlich konstant ist, müssen Sie h als Funktion der Zeit ableiten. Wenn also der See eine Anfangsbreite, -länge und -höhe hat, die einem Anfangsvolumen V entsprechen, nimmt dieses Volumen mit einer Rate von Q ab. Sie können also einfach aus der Geometrie die Höhenänderung als Funktion von Q berechnen, was ist selbst eine Funktion der Höhe. Es ist eine in sich konsistente Lösung.
Ah richtig, aber da der See ziemlich groß ist, kann ich davon ausgehen, dass h dann die ganze Zeit konstant ist. Ich habe die Formel integriert, die ähnlich wie Rodrigos Lösung ausfällt; aber es gibt mir falsche Werte (ich habe ein Diagramm mit ungefähren Werten). Verpasse ich etwas? Ich kann Ihnen die Lösung geben, wenn Sie möchten. Vielen Dank für deine Hilfe, wirklich!
@Eric Sie sollten diese Details in Ihrer ursprünglichen Frage posten. Außerdem vernachlässige ich die Viskosität - wenn das eine Annahme ist, die Sie bei Ihrer Hausaufgabe machen dürfen, dann sollten Sie diese Gleichungen (Rodrigos) in die richtige Richtung bringen.

Ich habe die Darcy-Formel zusammen mit den folgenden Formeln für eine schnelle numerische Lösung verwendet (nur wenige Iterationen erforderlich)

  1. H F = Δ P ρ G
  2. F = D A R C j ( R e )
  3. H F = F L D ( v 2 2 G )
  4. Löse oben nach v
  5. R e = ρ D v μ
  6. Gehen Sie zu Schritt 2 bis F konvergiert gegen einen Wert.
Bei Schritt 1 haben Sie Δ P , aber das habe ich nicht (wie in der Frage angegeben), oder habe ich Ihren ersten Schritt falsch verstanden?
Eigentlich Δ P = ρ G H So H F = H Genau genommen. Dies ist der Treiber (Antriebskraft) der Bewegung. Was mich beschäftigt ist die L was die Länge der Öffnung ist. Ich bin mir nicht sicher, ob es hier definiert ist.
Mit Länge meinst du Länge "in das Papier"? Angenommen, das Loch ist quadratisch, was ist D Dann? Der Durchmesser, schätze ich, aber wie gilt das für ein quadratisches Loch? Auch, was ist die Initiale R e ?
@Eric L ist die Länge Ihres Rohrs - dies gilt nur, wenn an der Öffnung (nicht in der Seite) ein Rohr angebracht ist. L liegt in Strömungsrichtung. Der R e ist die Reynoldszahl, die die Viskosität berücksichtigt. Dies erfasst die gleiche Physik wie die C Koeffizient in Rodrigos Gleichungen.
Ah stimmt, hab dich. Ich fühle mich ein bisschen dumm, nein, aber wie berechne ich in der ersten Iteration die Reibungskonstante? F wenn ich die nicht kenne R e Wert bis Schritt 5)? Es muss einen anfänglichen Re-Wert geben, oder?
Es spielt keine Rolle. Wählen F = 1 zu beginnen und von dort aus zu iterieren.

Das NCEES: FE-Referenzhandbuch enthält in seinem Abschnitt über Strömungsmechanik einige gute Materialien zum Flüssigkeitsfluss durch eine untergetauchte Öffnung. Sie können online danach suchen. NCEES stellt Ihnen kostenlos eine zur Verfügung.

Aus einem Wolfram-Artikel erhalten wir die vereinfachte Bernoulli-Gleichung:

Q = A C 2 G H

Wo

  • Q : die Durchflussrate ( M 3 / S )
  • A : die Fläche des Lochs ( M 2 )
  • C : Durchflusskoeffizient (dimensionslos)
  • G : die Erdbeschleunigung ( M / S 2 )
  • H : die Tiefe des Lochs ( M )

Das gilt für ein ausreichend kleines Loch, aber da Ihr Loch groß sein kann, müssen wir die Integralrechnung verwenden. Außerdem denke ich, dass der Durchflusskoeffizient für ein großes Loch auf 1 eingestellt werden kann. Und die Fläche des Lochs kann aus der Breite des Lochs mal der Höhe berechnet werden (unter der Annahme eines quadratischen Lochs).

So

Q = H D H 2 G j w D j = H D H w 2 G 2 j D j = w 2 G H D H 2 j D j = w 2 G [ 2 3 j 3 ] H D H = 2 3 w 2 G [ j 3 ] H D H = 2 3 w 2 G ( H 3 ( H D ) 3 )

Versuchen Sie, die Bitte des OP zu berücksichtigen: "Ich sage nicht: "Bitte geben Sie mir die Lösung, ich bin faul". Ich möchte es selbst herausfinden." Lasst uns darauf abzielen zu gefallen...
@ user3814483: Ok ... Ich bin mir nicht sicher, ob meine Gleichungen korrekt oder überhaupt für das Problem geeignet sind ... ich entwickle nur die Idee ...
Nein, das ist eigentlich in Ordnung, es ist komplizierter als ich dachte, danke. Ich habe es versucht, aber es hat mir nicht ganz die richtigen Zahlen gegeben, aber ich werde ein bisschen damit spielen. Vielen Dank @rodrigo
@Eric: Eigentlich würde ich erwarten, dass der tatsächliche Wert weit unter meinen Vorhersagen liegt. Erstens, weil echt C wäre niedriger; Zweitens, weil das Wasser, das oben aus dem Loch kommt, den Fluss aus dem Boden des Lochs behindert ... Es sei denn, es gibt natürlich ein Rohr, aber dann sind die Dinge noch komplizierter.
Ah hab dich. Vielen Dank für Ihre Hilfe! Ich habe eigentlich einen vorgegebenen Durchflusskoeffizienten, aber die Werte sind immer noch zu hoch. Ich denke, das hängt mit dem von dir erwähnten 2) zusammen.